整除规则
2、4、8 和 16 的整除规则:
若一个数的最后一位能被 \( 2\left( {2}^{1}\right) \) 整除,则该数能被 2 整除。若一个数的最后两位能被 4 整除,则该数能被 4 整除 \( \left( {2}^{2}\right) \) 。
若一个数的最后三位能被 8 整除,则该数能被 8 整除(2*)。
若一个数的最后四位能被 16 整除,则该数能被 16 整除(2*)。
5、25、125 和 225 的整除规则:
若一个数的最后一位能被 \( 5\left( {5}^{1}\right) \) 整除,则该数能被 5 整除。若一个数的最后两位组成的数能被 \( {25}\left( {5}^{2}\right) \) 整除,则该数能被 25 整除。
若一个数的最后三位组成的数能被 \( {125}\left( {5}^{3}\right) \) 整除,则该数能被 125 整除。
若一个数的最后四位组成的数能被 \( {625}\left( {5}^{4}\right) \) 整除,则该数能被 625 整除。
3 和 9 的整除规则
若一个数的各位数字之和能被 3 整除,则该数能被 3 整除。若一个数的各位数字之和能被 9 整除,则该数能被 9 整除。
7、11 和 13 的整除规则:
(1) 若将该数的最后一位数字乘以 2,再用其余部分减去所得结果,且差能被 7 整除,则原数能被 7 整除。如有必要,可对该差值再次应用此规则。
(2) 判断一个数能否被 11 整除:将奇数位数字相加,记为“x”;将偶数位数字相加,记为“y”。求 \( x \) 与 \( y \) 的正差值。若差值为 0 或 11 的倍数,则原数是 11 的倍数。
1. 基础知识
“能被整除”指“一个数除以另一个数所得结果为整数”。“能被整除”与“可被整除”含义相同。
表达式 \( \overline{abc},\underline{abc} \) 和 \( {abc} \) 相同。 \( \overline{abc} = \underline{abc} = {100a} + {10b} + c \) 。它们表示一个三位数,例如 \( \overline{234} = \underline{234} = {234} \) 。
从1到 \( n \) (含)之间能被 \( d \) 整除的正整数的个数是
\[ \left\lfloor \frac{n}{d}\right\rfloor \]
小于 \( n \) 且能被 \( d \) 整除的正整数的个数是 \( \left\lfloor \frac{n - 1}{d}\right\rfloor \) 。
\( \lfloor x\rfloor \) 称为下取整函数(floor function)。每当我们看到此符号时,取不超过 \( x \) 的最大整数值。
示例: \( \lfloor {3.14}\rfloor = 3,\lfloor {2.7}\rfloor = 2 \) 和 \( \lfloor {9.9}\rfloor = 9 \) 。
两事件的并集公式
集合 \( A \) 与 \( B \) 的并集,记作 \( A \cup B \) ,是所有属于 \( A \) 或 \( B \) 的元素的集合。
\[ n\left( {A \cup B}\right) = n\left( A\right) + n\left( B\right) - n\left( {A \cap B}\right) \]
三事件的并集公式 集合 \( A, B \) 和 \( C \) 的并集,记作 \( A \cup B \cup C \) ,是所有属于 \( A \) 或 \( B \) 或 \( C \) 的元素的集合。
\[ n\left( {A \cup B \cup C}\right) = n\left( A\right) + n\left( B\right) + n\left( C\right) \]
\[ - n\left( {A \cap B}\right) - n\left( {B \cap C}\right) - n\left( {C \cap A}\right) + n\left( {A \cap B \cap C}\right) \]
(3) 删除该数的最后一位,然后用剩余数减去被删数字的9倍。若所得结果能被13整除,则原数也能被13整除。如有必要,可重复此规则。
(4) 若该数最后三位与其余部分之差的绝对值能被7、11或13整除,则该数分别能被7、11或13整除。
6、10、12、14、15、18、24和36的整除规则:
若一个数同时能被2和3整除,则它能被6整除。
若一个数同时能被2和5整除,则它能被10整除。
若一个数同时能被3和4整除,则它能被12整除。
若一个数同时能被2和7整除,则它能被14整除。
若一个数同时能被3和5整除,则它能被15整除。
一个数能被18整除,当且仅当它能同时被2和9整除。
一个数能被24整除,当且仅当它能同时被3和8整除。
一个数能被36整除,当且仅当它能同时被4和9整除。
注:若一个数能被两个互质的数整除,则它必能被这两个数的乘积整除。“互质”指两数除1外无其他公因数。例如,3与4互质。
2. 解题技巧
(1). 利用整除规则
例1. 在前1000个正整数中,有多少个数不能被2、3、4、5、6全部整除?
(A) 983 (B) 17 (C) 984 (D) 753 (E) 617
解:(C)。
2、3、4、5、6的最小公倍数为60。能被2、3、4、5、6同时整除的数都是60的整数倍。共有 \( \left\lfloor \frac{1000}{60}\right\rfloor = {16} \) 个这样的数。因此答案为 \( {1000} - {16} = {984} \) 。
例2. 从1000到10000之间随机选取一个回文数,求其能被11整除的概率。
(A) \( 1/{10} \) (B) \( 5/9 \) (C) \( 3/7 \) (D) \( 5/6 \) (E) 1
解:(E)。
每个四位回文数可表示为 \( {abba} \) 。我们发现 \( a + b - (b + \) \( a) = 0 \) 。根据11的整除规则,该回文数必能被11整除,故所求概率为1。
例3. 若三位数 \( {78N} \) 能被4整除,则 \( N \) 有多少种可能的取值?
(A) 3 (B) 6 (C) 5 (D) 4 (E) 8
答案:A。
我们有五个末位为偶数的数:80、82、84、86 和 88。其中只有 80、84 和 88 能被 4 整除,因此 \( N \) 有 3 个取值:0、4 和 8。
例 4. 一个三位整数恰好包含数字 3、4、5 各一次。该整数能被 5 整除的概率是多少?
(A) \( 1/6 \) (B) \( 1/3 \) (C) \( 1/2 \) (D) \( 2/3 \) (E) \( 5/6 \)
答案:(B)。
该数末位等可能为 3、4 或 5。只有当末位为 5 时,该数才能被 5 整除,因此概率为 \( 1/3 \) 。
例 5. 六位数 \( \overline{713EF5} \) 能被 125 整除。这样的六位数共有多少个?
(A) 0 (B) 1 (C) 2 (D) 3 (E) 4
答案:(E)。
(713125、713375、713625 和 713875)。
若给定数能被 125 整除,则 \( {125},\overline{EF5} \) 应能被 125 整除。
\( {125} \times 1 = {125},{125} \times 3 = {375},{125} \times 5 = {625} \) ,以及 \( {125} \times 7 = {875} \) 。
例 6. 求数字 \( d \) 的最小可能值,使得 \( {437},{d03} \) 能被 9 整除。
。
(A) 1 (B) 2 (C) 3 (D) 4 (E) 5
答案:(A)。
\( 4 + 3 + 7 + d + 0 + 3 = {17} + d\; \Rightarrow \;d = 1 \) (唯一可能的值)。
例7. 哪个数不能被 \( 7?{616},{567},{798} \) 或878整除?
(A)616 (B)567 (C)798 (D)878 (E)无
解答:(D)。
616: \( {61} - 6 \times 2 = {61} - {12} = {49} \) (可被7整除)
567: \( {56} - 7 \times 2 = {56} - {14} = {42} \) (可被7整除)
798: \( {79} - 8 \times 2 = {79} - {16} = {63} \) (可被7整除)
878: \( {87} - 8 \times 2 = {87} - {16} = {71} \) (不能被7整除)。
例8. 若四位数 \( {273X} \) 能被12整除, \( {273X} \) 的值
是多少?
(A)3 (B)6 (C)5 (D)4 (E)8
解答:(B)。
四位数 \( {273X} \) 能被3和4整除。
\( 2 + 7 + 3 + X = {12} + \mathrm{X} \) 必须能被3整除,因此 \( X \) 可以是0、3、6、9。另一方面, \( X \) 必须为偶数,才能使该数被4整除,因此我们有 \( X = 0 \) 或6。 \( {3X} \) 也必须被4整除,所以 \( X = 6 \) 是唯一可行的值。
例9. 找出不同的数字 \( A \) 和 \( B \) ,使得 \( {A4}\underline{A4}B \) 尽可能大且能被36整除。写出该数。
(A)5472 (B)6471 (C)5470 (D)3474 (E)6470
解答:(A)。
\( \underline{A47B} \) 能被36整除,也能被9和4整除。
\( A + 4 + 7 + B = A + B + {11} \Rightarrow A + B = 7 \) 和 \( A + B = {16} \)
因为 \( A \) 和 \( B \) 互不相同且 \( B \) 必须为偶数, \( A + B = 7 \) 。
\( \underline{7B} \) 需要能被4整除,且我们希望 \( \underline{A47B} \) 取最大值。
\( A = 5 \) 和 \( B = 2.\underline{A47B} = {5472} \) 。
例10. 设 \( n \) 为一个四位数,当 \( n \) 除以100时,设 \( q \) 和 \( r \) 分别为商和余数。有多少个 \( n \) 使得 \( q \) \( + r \) 能被11整除?
(A) 909 (B) 811 (C) 812 (D) 819 (E) 890
解答:(D)。
注意 \( n = {100q} + r = q + r + {11} \times 9 \times q \) 。
若 \( n \) 能被 \( {11}, q + r \) 整除,则 \( {11}, q + r \) 必须能被11整除。
答案是 \( \left\lfloor \frac{9999}{11}\right\rfloor - \left\lfloor \frac{999}{11}\right\rfloor = {909} - {90} = {819} \) 。
例11. 若从所有各位数字之和等于34的四位数中随机选取一个数,该数能被11整除的概率是多少?
(A) \( 1/5 \) (B) \( 2/5 \) (C) \( 3/5 \) (D) \( 1/{11} \) (E) \( 1/{15} \)
解答:(B)。
由于十进制下最大数字为9,各位数字之和最多为 \( 4 \times 9 = {36} \) 。
给定的和34比36少2,可通过以下方式得到:
(1) 其中一个数字是7(比9少2),其余数字都是9。这种情况有 \( \frac{4!}{3! \times 1!} = 4 \) 种可能。这些数字是7999,9799,9979,9997。
(2) 其中两个数字是8(每个比9少1),另外两个数字是9。这种情况有 \( \frac{4!}{2! \times 2!} = 6 \) 种可能。这些数字是9988,8899,9898,8989,9889,8998。
接下来我们回忆,一个数能被11整除当且仅当其数字的交替和 \( {d}_{1} - {d}_{2} + {d}_{3} - {d}_{4} \) 能被11整除。我们发现这10个数中恰好有4个能被11整除,因此所求概率为 \( 4/{10} = 2/5 \) 。
例12.(2010 AMC 10 B)正整数 \( a, b \) 和 \( c \) 从集合 \( \{ 1,2,3,\ldots ,{2010}\} \) 中有放回地随机独立选取。 \( {abc} + {ab} + a \) 能被3整除的概率是多少?
(A) \( 1/3 \) (B) 29/81 (D) \( {11}/{27} \) (E) \( {13}/{27} \)
解答:(E)。
方法1(官方解答):
设 \( N = {abc} + {ab} + a = a\left( {{bc} + b + 1}\right) \) 。若 \( a \) 能被3整除,则 \( N \) 能被3整除。注意到2010能被3整除,因此 \( a \) 能被3整除的概率为 \( 1/3 \) 。若 \( a \) 不能被3整除,则 \( N \) 能被3整除当且仅当 \( {bc} + b + 1 \) 能被3整除。定义 \( {b}_{0} \) 和 \( {b}_{1} \) 使得 \( b = 3{b}_{0} + {b}_{1} \) 为整数且 \( {b}_{1} \) 等于0,1或2。注意到 \( {b}_{1} \) 的每个可能值概率均等。类似地定义 \( {c}_{0} \) 和 \( {c}_{1} \) 。则
\[ {bc} + b + 1 = \left( {3{b}_{0} + {b1}}\right) \left( {3{c}_{0} + {c1}}\right) + 3{b}_{0} + {b1} + 1 \]
\[ = 3\left( {3{b}_{0}{c}_{0} + {c}_{0}{b}_{1} + {c}_{1}{b}_{0} + {b}_{0}}\right) + {b}_{1}{c}_{1} + {b}_{1} + 1. \]
因此 \( {bc} + b + 1 \) 能被3整除当且仅当 \( {b}_{1} = 1 \) 和 \( {c}_{1} = 1 \) ,或 \( {b}_{1} = 2 \) 和 \( {c}_{1} = 0 \) 。这种情况发生的概率为 \( \frac{1}{3} \cdot \frac{1}{3} + \frac{1}{3} \cdot \frac{1}{3} = \frac{2}{9} \) 。
因此所求概率为 \( \frac{1}{3} + \frac{2}{9} \cdot \frac{2}{9} = \frac{13}{27} \) 。
方法2(我们的解法):
\( {abc} + {ab} + a = a\left( {{bc} + b + 1}\right) \) 。我们看到若 \( a \) 能被3整除,则 \( N \) 能被3整除。注意到2010能被3整除,因此 \( a \) 能被3整除的概率为 \( 1/3 \) 。
若 \( a \) 不能被3整除,则 \( {abc} + {ab} + a \) 能被3整除当且仅当 \( {bc} + b + 1 \) 能被3整除。
\( {bc} + b + 1 \equiv 0{\;\operatorname{mod}\;3} \Rightarrow \;{bc} + b \equiv 2{\;\operatorname{mod}\;3}\; \Rightarrow \;b\left( {c + 1}\right) \equiv 2{\;\operatorname{mod}\;3} \)
我们分两种情况:
情况1:
\( b \equiv 2{\;\operatorname{mod}\;3} \)
\( c \equiv 3{\;\operatorname{mod}\;3} \)
\( b \) 的取值数量为 \( m \) 和 \( {2010} = 2 + \left( {m - 1}\right) \times 3\; \Rightarrow m = {670} \) 。
出现这些取值的概率为 \( {670}/{2010} = 1/3 \) 。
\( c \) 的取值数量为 \( n \) 和 \( {2010} = 3 + \left( {n - 1}\right) \times 3\; \Rightarrow m = {670} \) 。
出现这些取值的概率为 \( {670}/{2010} = 1/3 \) 。
因此在情况1中,我们得到概率 \( \frac{1}{3} \cdot \frac{1}{3} = \frac{1}{9} \) 。
情况2:
\( b \equiv 1{\;\operatorname{mod}\;3} \)
\( c \equiv 1{\;\operatorname{mod}\;3} \)
\( b \) 的取值数量为 \( s \) 和 \( {2010} = 1 + \left( {s - 1}\right) \times 3 \Rightarrow s = {670} \) 。
出现这些取值的概率为 \( {670}/{2010} = 1/3 \) 。
\( c \) 的取值数量为 \( t \) 和 \( {2010} = 1 + \left( {t - 1}\right) \times 3\; \Rightarrow t = {670} \) 。
出现这些取值的概率为 \( {670}/{2010} = 1/3 \) 。
因此在情况2中,我们得到概率 \( \frac{1}{3} \cdot \frac{1}{3} = \frac{1}{9} \) 。
于是 \( {bc} + b + 1 \) 能被3整除的概率为 \( \frac{1}{9} + \frac{1}{9} = \frac{2}{9} \)
我们有以下情形:
(1) \( a \) 能被3整除,而 \( {bc} + b + 1 \) 不能被3整除;
(2) \( a \) 不能被3整除,而 \( {bc} + b + 1 \) 能被3整除;
(3) \( a \) 能被3整除,且 \( {bc} + b + 1 \) 也能被3整除。
最终答案为 \( \frac{1}{3} \times \frac{7}{9} + \frac{2}{3} \times \frac{2}{9} + \frac{1}{3} \times \frac{2}{9} = \frac{13}{27} \) 。
(2). 用维恩图(Venn Diagram)讨论整除性
例13. 在1到300(含)之间有多少个正整数同时是3和4的倍数?
(A) 100 (B) 75 (C) 50 (D) 25 (E) 18 解答:(D). \( \left\lfloor \frac{300}{3 \times 4}\right\rfloor = {25} \)
例14. 在1到300(含)之间有多少个正整数同时是3、4和5的倍数?
(A) 25 (B) 20 (C) 15 (D) 5 (E) 1
解答:(D).
\[ \left\lfloor \frac{300}{3 \times 4 \times 5}\right\rfloor = 5 \]
例15. 在1到300(含)之间有多少个正整数是3或4的倍数?
(A) 25 (B) 75 (C) 100 (D) 150 (E) 168
解答:(D).
\( n\left( {A \cup B}\right) = n\left( A\right) + n\left( B\right) - n\left( {A \cap B}\right) \)
\( \left\lfloor \frac{300}{3}\right\rfloor + \left\lfloor \frac{300}{4}\right\rfloor - \left\lfloor \frac{300}{3 \times 4}\right\rfloor = {100} + {75} - {25} = {150} \)
例16. 求1到300(含)之间既不被7整除也不被8整除的整数个数。
(A) 300 (B) 42 (C) 37 (D) 74 (E) 226
解答:(E).
设 \( A \) 为1到300中能被7整除的整数集合, \( B \) 为1到300中能被8整除的整数集合。小于或等于300且能被7或8整除的正整数个数即为集合 \( A \) 与集合 \( B \) 的并集,可按如下方式计算
\( n\left( {A \cup B}\right) = n\left( A\right) + n\left( B\right) - n\left( {A \cap B}\right) = \left\lfloor \frac{300}{7}\right\rfloor + \left\lfloor \frac{300}{8}\right\rfloor - \left\lfloor \frac{300}{7 \times 8}\right\rfloor = {42} + {37} - 5 = {74} \)
\( {300} - n\left( {A \cup B}\right) = {300} - {74} = {226} \) .
例17. 在1到300(含)之间有多少个正整数是3、4或5的倍数?
(A) 175 (B) 180 (C) 135 (D) 60 (E) 160
解答:(B)。
\( n\left( {A \cup B \cup C}\right) = n\left( A\right) + n\left( B\right) + n\left( C\right) \)
\( - n\left( {A \cap B}\right) - n\left( {B \cap C}\right) - n\left( {C \cap A}\right) + n\left( {A \cap B \cap C}\right) \)
\( {100} + {75} + {60} - \left( {{25} + {15} + {20}}\right) + 5 = {180}. \)
例18. 在1到300(含)之间,有多少个正整数是3或5的倍数但不是4的倍数?
(A) 175 (B) 125 (C) 105 (D) 180 (E) 75
解答:(C)。
\( \left\lfloor \frac{300}{3}\right\rfloor + \left\lfloor \frac{300}{4}\right\rfloor + \left\lfloor \frac{300}{5}\right\rfloor - \left\lfloor \frac{300}{3 \times 4}\right\rfloor - \left\lfloor \frac{300}{3 \times 5}\right\rfloor - \left\lfloor \frac{300}{4 \times 5}\right\rfloor + \left\lfloor \frac{300}{3 \times 4 \times 5}\right\rfloor = {100} + {75} - {25} = {150} \)
\[ = {100} + {75} + {60} - {25} - {20} - {15} + 5 = {180} \]
\[ \left\lfloor \frac{300}{4}\right\rfloor = {75} \]
所以答案是 \( {180} - {75} = {105} \) 。
例19.(2005 AMC 10B)在1到2005之间,有多少个数是3或4的整数倍但不是12的倍数?
(A) 501 (B) 668 (C) 835 (D) 1002 (E) 1169
解答:(C)。
方法1(官方解答):
在1到2005之间,有 \( \left\lfloor \frac{2005}{3}\right\rfloor = {668} \) 个 \( 3,\left\lfloor \frac{2005}{4}\right\rfloor = {501} \) 的倍数
4的倍数有 \( \left\lfloor \frac{2005}{12}\right\rfloor = {167} \) 个,12的倍数有 \( \left\lfloor \frac{2005}{12}\right\rfloor = {167} \) 个。因此,在1到2005之间,是3或4的整数倍但不是12的倍数的数共有 \( \left( {{668} - {167}}\right) + ({501} \) \( - {167}) = {835} \) 个。
方法2(我们的方法):
设 \( A \) 为3的倍数的整数个数, \( B \) 为4的倍数的整数个数, \( C \) 为12的倍数的整数个数。答案为 \( n\left( {A \cup B \cup C}\right) - n\left( C\right) \) 。
\[ n\left( {A \cup B \cup C}\right) - n\left( C\right) = n\left( A\right) + n\left( B\right) + n\left( C\right) \]
\[ - n\left( {A \cap B}\right) - n\left( {B \cap C}\right) - n\left( {C \cap A}\right) + n\left( {A \cap B \cap C}\right) - n\left( C\right) \]
\[ = \left\lfloor \frac{2005}{3}\right\rfloor + \left\lfloor \frac{2005}{4}\right\rfloor + \left\lfloor \frac{2005}{12}\right\rfloor - \left\lfloor \frac{2005}{\operatorname{lcm}\left( {3,4}\right) }\right\rfloor - \left\lfloor \frac{2005}{\operatorname{lcm}\left( {3,{12}}\right) }\right\rfloor - \left\lfloor \frac{2005}{\operatorname{lcm}\left( {4,{12}}\right) }\right\rfloor \]
\[ + \left\lfloor \frac{2005}{\operatorname{lcm}\left( {3,4,{12}}\right) }\right\rfloor - \left\lfloor \frac{2005}{12}\right\rfloor \]
\[ = \left\lfloor \frac{2005}{3}\right\rfloor + \left\lfloor \frac{2005}{4}\right\rfloor + \left\lfloor \frac{2005}{12}\right\rfloor - \left\lfloor \frac{2005}{12}\right\rfloor - \left\lfloor \frac{2005}{12}\right\rfloor - \left\lfloor \frac{2005}{12}\right\rfloor + \left\lfloor \frac{2005}{12}\right\rfloor - \left\lfloor \frac{2005}{12}\right\rfloor \]
\[ = \left\lfloor \frac{2005}{3}\right\rfloor + \left\lfloor \frac{2005}{4}\right\rfloor - 2 \times \left\lfloor \frac{2005}{12}\right\rfloor = {668} + {501} - 2 \times {167} = {835}. \]
例20.(2001 AMC 10B 第25题)在不超过2001的正整数中,有多少个数是3或4的倍数但不是5的倍数?(A) 768(B) 801 (C) 934 (D) 1067 (E) 1167 解答:(B)。方法1(官方解答):对于不超过2001的整数,有 \( \lfloor {2001}/3\rfloor = {667} \) 个3的倍数和 \( \lfloor {2001}/4\rfloor = {500} \) 个4的倍数。这 \( {667} + {500} = {1167} \) 个数中包含了 \( \lfloor {2001}/{12}\rfloor = {166} \) 个12的倍数两次,因此共有 \( {1167} - {166} = {1001} \) 个3或4的倍数。我们还需减去5的倍数的个数,于是从这组数中减去 \( \lfloor {2001}/{15}\rfloor = {133} \) 个15的倍数和 \( \lfloor {2001}/{20}\rfloor = {100} \) 个20的倍数,因为它们既是5和3的倍数,也是5和4的倍数。然而,在这一减法过程中, \( \lfloor {2001}/{60}\rfloor = {33} \) 个60的倍数被减去了两次,因此必须重新加回。满足条件的整数个数为 \( {1001} - {133} - {100} + {33} = {801} \) 。
方法2(我们的解答):
设圆 \( A \) 表示能被3整除的数集,圆 \( B \) 表示能被5整除的数集,圆 \( C \) 表示能被4整除的数集。
我们的任务是求下图中的阴影区域面积。为此,先求集合 \( A, B \) 与 \( C \) 的并集,再从集合 \( B \) 中减去该并集,得到最终结果。
\[ \left\lfloor \frac{2001}{3}\right\rfloor + \left\lfloor \frac{2001}{4}\right\rfloor + \left\lfloor \frac{2001}{5}\right\rfloor - \left\lfloor \frac{2001}{3 \times 4}\right\rfloor - \left\lfloor \frac{2001}{3 \times 5}\right\rfloor - \left\lfloor \frac{2001}{4 \times 5}\right\rfloor \]
\[ + \left\lfloor \frac{2001}{3 \times 4 \times 5}\right\rfloor - \left\lfloor \frac{2001}{5}\right\rfloor \]
\[ = \left\lfloor \frac{2001}{3}\right\rfloor + \left\lfloor \frac{2001}{4}\right\rfloor - \left\lfloor \frac{2001}{3 \times 4}\right\rfloor - \left\lfloor \frac{2001}{3 \times 5}\right\rfloor - \left\lfloor \frac{2001}{4 \times 5}\right\rfloor + \left\lfloor \frac{2001}{3 \times 4 \times 5}\right\rfloor \]
\[ = {667} + {500} - {11} - {133} - {100} + {33} = {801}. \]
例21. 从1到300(含)之间所有能被3、4或5整除的正整数中随机选取一个正整数。求选出的正整数不能被5整除的概率。答案用普通分数表示。
(A) \( 2/3 \) (B) \( 3/{10} \) (C) \( 1/5 \) (D) \( 1/2 \) (E) \( 8/{25} \)
解答:(A)。
\[ \left\lfloor \frac{300}{3}\right\rfloor + \left\lfloor \frac{300}{4}\right\rfloor + \left\lfloor \frac{300}{5}\right\rfloor - \left\lfloor \frac{300}{3 \times 4}\right\rfloor - \left\lfloor \frac{300}{3 \times 5}\right\rfloor - \left\lfloor \frac{300}{4 \times 5}\right\rfloor + \left\lfloor \frac{300}{3 \times 4 \times 5}\right\rfloor \]
\[ = {100} + {75} + {60} - {25} - {20} - {15} + 5 = {180} \]
\[ \left\lfloor \frac{300}{5}\right\rfloor = {60} \]
\[ {180} - {60} = {120}\text{.} \]
概率为 \( P = \frac{120}{180} = \frac{2}{3} \) 。
例22 从1到300(含)之间所有能被3、4或5整除的正整数中随机选取一个正整数。求选出的正整数仅能被5整除的概率。答案用普通分数表示。
(A) \( 1/6 \) (B) \( 1/9 \) (C) \( 2/9 \) (D) \( 1/2 \) (E) \( 3/8 \)
解答:(A)。
\[ \left\lfloor \frac{300}{3}\right\rfloor + \left\lfloor \frac{300}{4}\right\rfloor + \left\lfloor \frac{300}{5}\right\rfloor - \left\lfloor \frac{300}{3 \times 4}\right\rfloor - \left\lfloor \frac{300}{3 \times 5}\right\rfloor - \left\lfloor \frac{300}{4 \times 5}\right\rfloor + \left\lfloor \frac{300}{3 \times 4 \times 5}\right\rfloor \]
\[ = {100} + {75} + {60} - {25} - {20} - {15} + 5 = {180} \]
\( \left\lfloor \frac{300}{3}\right\rfloor + \left\lfloor \frac{300}{4}\right\rfloor - \left\lfloor \frac{300}{3 \times 4}\right\rfloor = {100} + {75} - {25} = {150} \)
\( {180} - {150} = {30} \) .
概率为 \( P = \frac{30}{180} = \frac{1}{6} \) 。
例23.(2003 AMC 10A)集合 \( \{ 1,2,3,\ldots ,{100}\} \) 中的整数能被2整除且不能被3整除的概率是多少?
(A) \( 1/6 \) (B) \( {33}/{100} \) (C) \( {17}/{50} \) (D) \( 1/2 \) (E) \( {18}/{25} \)
解答:(C)。
方法1(官方解答):
在能被2整除的 \( {100}/2 = {50} \) 个整数中,有 \( \left\lfloor \frac{100}{6}\right\rfloor = {16} \) 个
同时能被2和3整除。因此,能被2整除但不能被3整除的有 \( {50} - {16} = {34} \) 个,以及 \( {34}/{100} = {17}/{50} \) 。
方法2(我们的解法):
显然,我们分数的分母为100,即集合中整数的总数。为确定分子,必须找出1到100中能被2整除且不能被3整除的整数个数。
设 \( A \) 为1到100中能被2整除的整数集合, \( B \) 为1到100中能被3整除的整数集合。
我们的任务是求下图中的阴影区域:
为求阴影区域,首先计算 \( {AB} \) 的并集
\( n\left( {A \cup B}\right) = n\left( A\right) + n\left( B\right) - n\left( {A \cap B}\right) = \left\lfloor \frac{100}{2}\right\rfloor + \left\lfloor \frac{100}{3}\right\rfloor - \left\lfloor \frac{100}{2 \times 3}\right\rfloor = {50} + {33} - {16} = {67} \)
接着,减去集合 \( B : {67} - \left\lfloor \frac{100}{3}\right\rfloor = {67} - {33} = {34} \) 。
因此,共有34个能被2整除且不能被3整除的整数。
最终答案即为 \( \frac{34}{100} = \frac{17}{50} \) 。
例24. 在1到840(含)的整数中,有多少个整数是3和4的公倍数或5和6的公倍数,但不是7和8的公倍数?
(A) 79 (B) 28 (C) 14 (D) 8 (E) 5
解答:(A)。
问题要求计算1到840之间是12或30的倍数但不是56的倍数的整数个数。
设 \( A \) 为12的倍数的整数个数。
设 \( B \) 为30的倍数的整数个数。
设 \( C \) 为56的倍数的整数个数。
答案即为 \( n\left( {A \cup B \cup C}\right) - n\left( C\right) \) 。
\[ n\left( {A \cup B \cup C}\right) = n\left( A\right) + n\left( B\right) + - n\left( {A \cap B}\right) - n\left( {B \cap C}\right) - n\left( {C \cap A}\right) + n\left( {A \cap B \cap C}\right) \]
\[ \left\lfloor \frac{840}{12}\right\rfloor + \left\lfloor \frac{840}{30}\right\rfloor + \left\lfloor \frac{840}{56}\right\rfloor - \left\lfloor \frac{840}{\mathrm{{lcm}}\left( {{12},{30}}\right) }\right\rfloor - \left\lfloor \frac{840}{\mathrm{{lcm}}\left( {{12},{56}}\right) }\right\rfloor - \left\lfloor \frac{840}{\mathrm{{lcm}}\left( {{30},{56}}\right) }\right\rfloor + \left\lfloor \frac{840}{\mathrm{{lcm}}\left( {{12},{30},{56}}\right) }\right\rfloor \]
\[ = \left\lfloor \frac{840}{12}\right\rfloor + \left\lfloor \frac{840}{30}\right\rfloor + \left\lfloor \frac{840}{56}\right\rfloor - \left\lfloor \frac{840}{60}\right\rfloor - \left\lfloor \frac{840}{168}\right\rfloor - \left\lfloor \frac{840}{840}\right\rfloor + \left\lfloor \frac{840}{840}\right\rfloor \]
\[ = \left\lfloor \frac{840}{12}\right\rfloor + \left\lfloor \frac{840}{30}\right\rfloor + \left\lfloor \frac{840}{56}\right\rfloor - \left\lfloor \frac{840}{60}\right\rfloor - \left\lfloor \frac{840}{168}\right\rfloor . \]
\[ n\left( C\right) = \left\lfloor \frac{840}{56}\right\rfloor \]
\( n\left( {A \cup B \cup C}\right) - n\left( C\right) = \left\lfloor \frac{840}{12}\right\rfloor + \left\lfloor \frac{840}{30}\right\rfloor + \left\lfloor \frac{840}{56}\right\rfloor - \left\lfloor \frac{840}{60}\right\rfloor - \left\lfloor \frac{840}{168}\right\rfloor - \left\lfloor \frac{840}{56}\right\rfloor \)
\[ = \left\lfloor \frac{840}{12}\right\rfloor + \left\lfloor \frac{840}{30}\right\rfloor - \left\lfloor \frac{840}{60}\right\rfloor - \left\lfloor \frac{840}{168}\right\rfloor \]
\[ = {70} + {28} - {14} - 5 = {79}\text{.} \]
例25. \( K \) 是所有满足以下性质的自然数集合:集合中的每个元素既不被3整除也不被4整除;但若一个数能被3或4整除且同时能被5整除,则该数属于 \( K \) (如15,20,60,…)。求 \( K \) 的第 \( {79}^{\text{th }} \) 个最小元素。
(A) 127 (B) 130 (C) 131 (D) 133 (E) 135
解答:(D)。
方法一:
设 \( {a}_{79} = n \) 。
\[ {79} = n - n\left( {A \cup B}\right) + n\left( {A \cap C}\right) + n\left( {B \cap C}\right) - n\left( {A \cap B \cap C}\right) \]
\[ = n - \left\lfloor \frac{n}{3}\right\rfloor - \left\lfloor \frac{n}{4}\right\rfloor + \left\lfloor \frac{n}{3 \times 4}\right\rfloor + \left\lfloor \frac{n}{3 \times 5}\right\rfloor + \left\lfloor \frac{n}{4 \times 5}\right\rfloor - \left\lfloor \frac{n}{3 \times 4 \times 5}\right\rfloor \tag{1} \]
我们知道 \( x - 1 < \lfloor x\rfloor \leq x \) 。
于是 \( {79} < n - \left( {\frac{n}{3} - 1}\right) - \left( {\frac{n}{4} - 1}\right) + \frac{n}{3 \times 4} + \frac{n}{3 \times 5} + \frac{n}{4 \times 5} - \left( {\frac{n}{3 \times 4 \times 5} - 1}\right) = \frac{3}{5}n + 3 \) (2)
且 \( {79} > n - \frac{n}{3} - \frac{n}{4} + \left( {\frac{n}{3 \times 4} - 1}\right) + \left( {\frac{n}{3 \times 5} - 1}\right) + \left( {\frac{n}{4 \times 5} - 1}\right) - \frac{n}{3 \times 4 \times 5} = \frac{3}{5}n - 3 \) (3)
由(2)和(3)得 \( {126}\frac{2}{3} < n < {136}\frac{2}{3} \) 或 \( {127} \leq n \leq {136} \) 。
已知 \( n \) 既不是3的倍数也不是4的倍数(除非同时是5的倍数),因此 \( n \) 必为127,130,131,133,134,135之一。
将这些可能值代入(1),唯一满足条件的解为 \( n = \) 133。
方法二:
3、4、5的最小公倍数为60。
\[ {60} - \left\lfloor \frac{60}{3}\right\rfloor - \left\lfloor \frac{60}{4}\right\rfloor + \left\lfloor \frac{60}{3 \times 4}\right\rfloor + \left\lfloor \frac{60}{3 \times 5}\right\rfloor + \left\lfloor \frac{60}{4 \times 5}\right\rfloor - \left\lfloor \frac{60}{3 \times 4 \times 5}\right\rfloor = {36}. \]
在1到60之间, \( {a}_{n} \) 共有36项。
\[ {a}_{1} = 1,{a}_{2} = 2,{a}_{3} = 5,{a}_{4} = 7,{a}_{5} = {10},{a}_{6} = {11}{a}_{7} = {13},\ldots ,{a}_{36} = {60}\text{.} \]
我们知道 \( {79} = {36} \times 2 + 7 \) 。因此 \( {a}_{79} = {60} \times 2 + {a}_{7} = {120} + {13} = {133} \) 。
例26. \( K \) 是与105互质的所有自然数集合。求 \( K \) 的第 \( {142}^{\text{nd }} \) 个最小元素。
(A) 302 (B) 307 (C) 308 (D) 311 (E) 317
解答:(D)。
方法一:
\( {105} = 3 \times 5 \times 7. \)
设 \( {a}_{79} = n \) 。
\[ {142} = n - \left\lfloor \frac{n}{3}\right\rfloor - \left\lfloor \frac{n}{5}\right\rfloor - \left\lfloor \frac{n}{7}\right\rfloor + \left\lfloor \frac{n}{3 \times 5}\right\rfloor + \left\lfloor \frac{n}{3 \times 7}\right\rfloor + \left\lfloor \frac{n}{5 \times 7}\right\rfloor - \left\lfloor \frac{n}{3 \times 5 \times 7}\right\rfloor \tag{1} \]
我们知道 \( x - 1 < \lfloor x\rfloor \leq x \) 。
于是 \( {142} < n - \left( {\frac{n}{3} - 1}\right) - \left( {\frac{n}{5} - 1}\right) - \left( {\frac{n}{7} - 1}\right) + \frac{n}{3 \times 5} + \frac{n}{3 \times 7} + \frac{n}{5 \times 7} - \left( {\frac{n}{3 \times 5 \times 7} - 1}\right) = \frac{16}{35}n + 4 \) (2) 且 142 \( > n - \frac{n}{3} - \frac{n}{5} - \frac{n}{7} + \left( {\frac{n}{3 \times 5} - 1}\right) + \left( {\frac{n}{3 \times 7} - 1}\right) + \left( {\frac{n}{5 \times 7} - 1}\right) - \frac{n}{3 \times 4 \times 5} = \frac{16}{35}n - 3 \) (3)。由 (2) 和 (3) 得 \( {301}\frac{7}{8} < n < {317}\frac{3}{16} \) 或 \( {302} \leq n \leq {317} \) 。我们知道 \( n \) 与 105 互质,因此 \( n \) 必为 302、307、308、311、313、314、316、317 之一。
将 \( n = {317} \) 代入 (1),我们发现
\[ n - \left\lfloor \frac{n}{3}\right\rfloor - \left\lfloor \frac{n}{5}\right\rfloor - \left\lfloor \frac{n}{7}\right\rfloor + \left\lfloor \frac{n}{3 \times 5}\right\rfloor + \left\lfloor \frac{n}{3 \times 7}\right\rfloor + \left\lfloor \frac{n}{5 \times 7}\right\rfloor - \left\lfloor \frac{n}{3 \times 5 \times 7}\right\rfloor = {146}\text{, which is 4 more} \]
比 142 大。因此 \( n = {311} \) 应成立,确实如此,我们得到
\[ n - \left\lfloor \frac{n}{3}\right\rfloor - \left\lfloor \frac{n}{5}\right\rfloor - \left\lfloor \frac{n}{7}\right\rfloor + \left\lfloor \frac{n}{3 \times 5}\right\rfloor + \left\lfloor \frac{n}{3 \times 7}\right\rfloor + \left\lfloor \frac{n}{5 \times 7}\right\rfloor - \left\lfloor \frac{n}{3 \times 5 \times 7}\right\rfloor = {142}. \]
方法二:
\[ {105} = 3 \times 5 \times 7 \]
从 1 到 105 中与 105 互质的数的个数为
\[ {105} - \left\lfloor \frac{105}{3}\right\rfloor - \left\lfloor \frac{105}{5}\right\rfloor - \left\lfloor \frac{105}{7}\right\rfloor + \left\lfloor \frac{105}{3 \times 5}\right\rfloor + \left\lfloor \frac{105}{3 \times 7}\right\rfloor + \left\lfloor \frac{105}{5 \times 7}\right\rfloor - \left\lfloor \frac{105}{3 \times 5 \times 7}\right\rfloor \]
\[ = {105} - \left( {{35} + {21} + {15}}\right) + \left( {7 + 3 + 5}\right) - 1 = {48}\text{.} \]
于是 \( {a}_{1} = 1,{a}_{2} = 2,{a}_{3} = 4,\ldots ,{a}_{45} = {97},{a}_{46} = {101},{a}_{47} = {103} \) ,且 \( {a}_{48} = {104} \) 。
我们还有 \( {a}_{49} = {105} + 1,{a}_{50} = {105} + 2,{a}_{51} = {105} + 4,\ldots ,{a}_{96} = {105} + {104},\ldots \)
由于 \( {142} = {48} \times 2 + {46} \) ,因此 \( {a}_{142} = {105} \times 2 + {a}_{46} = {210} + {101} = {311} \) 。
3. 习题
问题 1. 在个位用哪个数字替换 \( a \) ,才能使 9867542a 被 11 整除?
(A) 3 (B) 6 (C) 9 (D) 4 (E) 8
问题2. 能被6整除的最大三位数是多少?
(A) 999 (B) 998 (C) 997 (D) 996 (E) 995
问题3. 当 \( x \) 取哪些数字时,7位数 \( {3xx6xx2} \) 能被4整除?
(A) 3 (B) 6 (C) 2 (D) 5 (E) 8
问题4. 要使数字437, b32能被3整除, \( b \) 可替换的最大数字是多少?
(A) 3 (B) 6 (C) 2 (D) 5 (E) 8
问题5. \( A \) 和 \( B \) 为非零数字,且 \( \underline{A468B05} \) 能被11整除,则 \( A + B \) 等于多少?
(A) 6 (B) 8 (C) 10 (D) 14 (E) 12
问题6. 一个四位数恰好使用数字1、2、3、4各一次,该数是4的倍数的概率是多少?
(A) \( 1/2 \) (B) \( 1/3 \) (C) \( 1/5 \) (D) \( 1/4 \) (E) \( 3/8 \)
问题7. 已知 \( m \) 和 \( n \) 为数字,使五位数 \( {m6},{79n} \) 能被72整除的所有 \( m \) 与 \( n \) 值之和是多少?
(A) 4 (B) 3 (C) 7 (D) 10 (E) 5
问题8. 求 \( x \) 的值,使四位数 \( {x15x} \) 能被18整除。
(A) 1 (B) 2 (C) 4 (D) 6 (E) 8
问题9. 要使 \( {a74a} \) 能被36整除, \( a \) 可以取何值?
(A) 2 (B) 4 (C) 6 (D) 8 (E) 9
问题10。设 \( n \) 为一个6位数,当 \( n \) 除以100时, \( \mathrm{q} \) 和 \( \mathrm{r} \) 分别为商和余数。有多少个 \( n \) 使得 \( q \) \( + r \) 能被11整除?
(A) 81818 (B) 81819 (C) 81820 (D) 81909 (E) 89909
问题11。在一个被分成 \( n \) 行、每行 \( n \) 个方格的正方形游戏棋盘上, \( k \) 个方格不位于棋盘边界。如果 \( k \) 是下面四个数之一, \( n \) 的可能值是多少?
(I) 10 (II) 25 (III) 34 (IV) 52
(A) 10 (B) 12 (C) 14 (D) 16 (E) 18
问题12。正整数 \( a, b \) 和 \( c \) 从集合 \( \{ 1,2,3,\ldots ,{2016}\} \) 中有放回地随机且独立地选取。 \( {abc} + {ab} + a \) 能被3整除的概率是多少?
(A) \( 1/3 \) (B) \( {29}/{81} \) (C) \( {31}/{81} \) (D) \( {11}/{27} \) (E) \( {13}/{27} \)
问题13。在1到300(含)之间有多少个正整数同时是4和5的倍数?
(A) 10 (B) 12 (C) 15 (D) 16 (E) 18
问题14。在1到2016(含)之间有多少个正整数同时是3、4和5的倍数?
(A) 33 (B) 55 (C) 100 (D) 134 (E) 168
问题15。在1到300(含)之间有多少个正整数是3或5的倍数?
(A) 133 (B) 140 (C) 150 (D) 134 (E) 168
问题16。不超过2100且既不被5也不被7整除的正整数有多少个?
(A) 660 (B) 1440 (C) 420 (D) 1340 (E) 1680
问题17。在1到2016(含)之间有多少个正整数是3、4或5的倍数?
(A) 1176 (B) 1210 (C) 907 (D) 1075 (E) 1209
问题18. 在1到300(含)之间,有多少个正整数是4和5的公倍数,但不是3的倍数?
(A) 180 (B) 80 (C) 105 (D) 150 (E) 75
问题19. 不超过2001的正整数中,有多少个是3或4的倍数,但不是12的倍数?
(A) 667 (B) 550 (C) 166 (D) 850 (E) 835
问题20. 不超过2016的正整数中,有多少个是3或4的倍数,但不是5的倍数?
(A) 672 (B) 807 (C) 504 (D) 168 (E) 1176
问题21. 在1到2015(含)之间,有多少个数是3或5的整数倍,但不是15的倍数?
(A) 403 (B) 671 (C) 835 (D) 268 (E) 806
问题22. 不超过300的正整数中,有多少个是3或4的倍数,但不是15的倍数?
(A) 20 (B) 143 (C) 163 (D) 75 (E) 7
问题23. 在集合 \( \{ 1,2,3,\ldots ,{1000}\} \) 中随机取一个整数,该整数能被2整除且不能被3整除的概率是多少?
(A) \( 1/6 \) (B) \( {33}/{100} \) (C) 167/500 (D) \( 1/2 \) (E) \( {18}/{25} \)
问题24. 在1到840之间,有多少个整数是3和5的公倍数,但不是4或7的倍数?
(A) 36 (B) 300 (C) 14 (D) 8 (E) 2
问题25. \( K \) 是自然数集合,其每个元素既不被3整除,也不被4整除。但若一个数能被
3 或 4 且能被 5 整除,这个数属于 \( K \) (如 15、20、60……)。求 \( K \) 中第 2009 小的数。
(A) 3345 (B) 3346 (C) 3347 (D) 3349 (E) 3350
问题 26。 \( K \) 是与 105 互质的自然数集合。求 \( K \) 中第 1000 小的数。
(A) 2176 (B) 2182 (C) 2183 (D) 2186 (E) 2189
4. 解答
问题 1。解答:(A)。
设 \( x = 9 + 6 + 5 + 2, y = 8 + 7 + 4 + a \) 。
\( x - y = \left( {9 + 6 + 5 + 2}\right) - \left( {8 + 7 + 4 + a}\right) = {22} - {19} - a = 3 - a.\; \Rightarrow \;a = 3 \) .
问题 2。解答:(D)。
该数必须同时被 2 和 3 整除,因此它是偶数且各位数字之和能被 3 整除。若该数为 \( {99x}, x = 6 \) 则成立。
问题 3。解答:(D)。
1、3、5、7 和 9。
两位数 \( {x2} \) 需能被 4 整除。满足条件的数有:12、32、52、72 和 92。问题 4。解答:(E)。 \( 4 + 3 + 7 + b + 3 + 2 = {19} + b \Rightarrow b = 2,5 \) ,以及 8。 \( b \) 的最大值为 8。问题 5。解答:(E)。设 \( x = A + 6 + B + 5, y = 4 + 8 + 0 \) 。 \( x - y = \left( {A + 6 + B + 5}\right) - \left( {4 + 8 + 0}\right) = {11} + B + A - {12} = B + A - 1 \Rightarrow \;B + A \) \( = {12} \) 。问题 6。解答:(D)。末两位组成的数必须能被 4 整除。末两位的可能情况如下:
\[ \begin{array}{ll} 1 & 2 \end{array} \]
\[ \text{2} \]
\[ \text{3 2} \]
对于所有三种情况,我们都能构造出两个这样的四位数。
因此答案为 \( 6/{24} = 1/4 \) 。
问题 7。解答:(E)。
\( {72} = 8 \times 9 \Rightarrow {m6},{79n} \) 必须同时被 8 和 9 整除。
当 \( {m6},{79n} \) 能被8整除时,三位数 \( {79n} \) 必须能被8整除,且 \( n \) 必须为偶数。由此得到 \( n = 2 \) 。
当 \( {m6},{79n} \) 能被9整除时, \( 9, m + 6 + 7 + 9 + 2 = m + {24} \) 必须能被9整除。注意到 \( 0 \leq m \leq 9 \) ,因此 \( m \) 必须为3。
\( m + n = 3 + 2 = 5 \) .
问题8。解答:(D)。
由于 \( \underline{x15x} \) 能被18整除,它必须同时能被2和9整除。
\( x + 1 + 5 + x = {2x} + 6\; \Rightarrow \;x = 6 \) (唯一可能的值)。
问题9。解答:(D)。
该数能被4和9整除。为了使 \( {a74a} \) 能被4整除, \( a \) 可以是4或8。为了使 \( \underline{a74a} \) 能被 \( 9,{2a} + 7 + 4 = {2a} + {11} \) 整除, \( 9,{2a} + 7 + 4 = {2a} + {11} \) 必须能被9整除。 \( a \) 的唯一可能值是8。
问题10。解答:(B)。
注意 \( n = {100q} + r = q + r + {11} \times 9 \times r \) 。
如果 \( n \) 能被11整除,则 \( {11}, q + r \) 必须能被11整除。
答案是 \( \left\lfloor \frac{999999}{11}\right\rfloor - \left\lfloor \frac{99999}{11}\right\rfloor = {90909} - {9090} = {81819} \) 。
问题11。解答:(C)。
不位于棋盘边界上的正方形数量为 \( {n}^{2} - {\left( n - 2\right) }^{2} = 4\left( {n - 1}\right) \) ,该数能被4整除。因此我们知道 \( k = {52} \) 。当 \( k = \) 为52时,即 \( 4\left( {n - 1}\right) = {52} \) 。解得 \( n : n = {14} \) 。
问题12。解答:(E)。
\( {abc} + {ab} + a = a\left( {{bc} + b + 1}\right) \) 。我们发现,如果 \( a \) 能被3整除,则 \( {abc} + {ab} + a \) 也能被3整除。注意到2016能被3整除,因此 \( a \) 能被3整除的概率为 \( 1/3 \) 。
如果 \( a \) 不能被3整除,那么当 \( {bc} + b + 1 \) 能被3整除时, \( {abc} + {ab} + a \) 能被3整除。 \( {bc} + b + 1 \equiv 0{\;\operatorname{mod}\;3} \Rightarrow \;{bc} + b \equiv 2{\;\operatorname{mod}\;3}\; \Rightarrow \;b\left( {c + 1}\right) \equiv 2{\;\operatorname{mod}\;3} \) 我们有两种情况:
情况1:
\( b \equiv 2{\;\operatorname{mod}\;3} \)
\( c \equiv 3{\;\operatorname{mod}\;3} \)
\( b \) 的取值个数为 \( m \) 和 \( {2016} = 2 + \left( {m - 1}\right) \times 3\; \Rightarrow m = {672} \) 。
取到这些值的概率为 \( {670}/{2010} = 1/3 \) 。
\( c \) 的取值个数为 \( n \) 和 \( {2016} = 3 + \left( {n - 1}\right) \times 3 \)
取到这些值的概率为 \( {672}/{2016} = 1/3 \) 。
因此在情况1中,我们得到概率 \( \frac{1}{3} \cdot \frac{1}{3} = \frac{1}{9} \) 。
情况2:
\( b \equiv 1{\;\operatorname{mod}\;3} \)
\( c \equiv 1{\;\operatorname{mod}\;3} \)
\( b \) 的取值个数为 \( s \) 和 \( {2016} = 1 + \left( {s - 1}\right) \times 3 \Rightarrow s = {672} \) 。
取到这些值的概率为 \( {672}/{2016} = 1/3 \) 。
\( c \) 的取值个数为 \( t \) 和 \( {2016} = 1 + \left( {t - 1}\right) \times 3\; \Rightarrow t = {672} \) 。
取到这些值的概率为 \( {672}/{2016} = 1/3 \) 。
因此在情况2中,我们得到概率 \( \frac{1}{3} \cdot \frac{1}{3} = \frac{1}{9} \) 。
于是 \( {bc} + b + 1 \) 能被3整除的概率为 \( \frac{1}{9} + \frac{1}{9} = \frac{2}{9} \)
我们有以下情形:
(1) \( a \) 能被3整除,而 \( {bc} + b + 1 \) 不能被3整除;
(2) \( a \) 不能被3整除,而 \( {bc} + b + 1 \) 能被3整除;
(3) \( a \) 能被3整除, \( {bc} + b + 1 \) 也能被3整除。
最终答案为 \( \frac{1}{3} \times \frac{7}{9} + \frac{2}{3} \times \frac{2}{9} + \frac{1}{3} \times \frac{2}{9} = \frac{13}{27} \) 。
问题13。
解答:(C)。
\[ \left\lfloor \frac{300}{4 \times 5}\right\rfloor = {15} \]
问题14。解答:(A)。
\[ \left\lfloor \frac{2016}{3 \times 4 \times 5}\right\rfloor = {33} \]
问题15。解答:(B)。
\[ \left\lfloor \frac{300}{3}\right\rfloor + \left\lfloor \frac{300}{5}\right\rfloor - \left\lfloor \frac{300}{3 \times 5}\right\rfloor = {100} + {60} - {20} = {140} \]
问题16。解答:(B)。
小于2100且能被5或7整除的正整数的个数为 \( \left\lfloor \frac{2100}{5}\right\rfloor + \left\lfloor \frac{2100}{7}\right\rfloor - \left\lfloor \frac{2100}{5 \times 7}\right\rfloor = {420} + {300} - {60} = {660}. \)
因此,所求答案为 \( {2100} - {660} = {1440} \) 。
问题17。解答:(B)。
\[ n\left( {A \cup B \cup C}\right) = n\left( A\right) + n\left( B\right) + n\left( C\right) \]
\[ - n\left( {A \cap B}\right) - n\left( {B \cap C}\right) - n\left( {C \cap A}\right) + n\left( {A \cap B \cap C}\right) \]
\[ = \left\lfloor \frac{2016}{3}\right\rfloor + \left\lfloor \frac{2016}{4}\right\rfloor + \left\lfloor \frac{2016}{5}\right\rfloor - \left\lfloor \frac{2016}{3 \times 4}\right\rfloor - \left\lfloor \frac{2016}{3 \times 5}\right\rfloor - \left\lfloor \frac{2016}{4 \times 5}\right\rfloor + \left\lfloor \frac{2016}{3 \times 4 \times 5}\right\rfloor \]
\[ = {672} + {504} + {403} - \left( {{168} + {134} + {100}}\right) + {33} = {1210}. \]
问题18。解答:(B)。
\[ \left\lfloor \frac{300}{3}\right\rfloor + \left\lfloor \frac{300}{4}\right\rfloor + \left\lfloor \frac{300}{5}\right\rfloor - \left\lfloor \frac{300}{3 \times 4}\right\rfloor - \left\lfloor \frac{300}{3 \times 5}\right\rfloor - \left\lfloor \frac{300}{4 \times 5}\right\rfloor + \left\lfloor \frac{300}{3 \times 4 \times 5}\right\rfloor = {100} + {75} - {25} = {150} \]
\[ = {100} + {75} + {60} - {25} - {20} - {15} + 5 = {180} \]
\[ \left\lfloor \frac{300}{3}\right\rfloor = {100} \]
所以答案为 \( {180} - {100} = {80} \) 。
问题19。解答:(E)。
方法1:设圆 \( A \) 表示能被3整除的数集,圆 \( B \) 表示能被4整除的数集。
对于本题,我们的任务是求阴影区域。
为求阴影区域,先求集合 \( A \) 与 \( B \) 的并集,再减去 \( A \) 与 \( B \) 的交集。
\[ \left\lfloor \frac{2001}{3}\right\rfloor + \left\lfloor \frac{2001}{4}\right\rfloor - \left\lfloor \frac{2001}{3 \times 4}\right\rfloor - \left\lfloor \frac{2001}{3 \times 4}\right\rfloor = {667} + {500} - {166} - {166} = {835}. \]
方法二:
\[ \left\lbrack \frac{2001}{3}\right\rbrack + \left\lbrack \frac{2001}{4}\right\rbrack + \left\lbrack \frac{2001}{12}\right\rbrack - \left\lbrack \frac{2001}{3 \times 4}\right\rbrack - \left\lbrack \frac{2001}{{LCM}\left( {3 \times {12}}\right) }\right\rbrack - \left\lbrack \frac{2001}{{LCM}\left( {4 \times {12}}\right) }\right\rbrack + \]
\[ \left\lfloor \frac{2001}{{LCM}\left( {3 \times 4 \times {12}}\right) }\right\rfloor - \left\lfloor \frac{2001}{12}\right\rfloor \]
\[ = \left\lfloor \frac{2001}{3}\right\rfloor + \left\lfloor \frac{2001}{4}\right\rfloor - \left\lfloor \frac{2001}{3 \times 4}\right\rfloor - \left\lfloor \frac{2001}{{LCM}\left( {3 \times {12}}\right) }\right\rfloor - \left\lfloor \frac{2001}{{LCM}\left( {4 \times {12}}\right) }\right\rfloor + \left\lfloor \frac{2001}{{LCM}\left( {3 \times 4 \times {12}}\right) }\right\rfloor \]
\[ = \left\lfloor \frac{2001}{3}\right\rfloor + \left\lfloor \frac{2001}{4}\right\rfloor - \left\lfloor \frac{2001}{3 \times 4}\right\rfloor - \left\lfloor \frac{2001}{12}\right\rfloor - \left\lfloor \frac{2001}{12}\right\rfloor + \left\lfloor \frac{2001}{12}\right\rfloor \]
\[ = \left\lfloor \frac{2001}{3}\right\rfloor + \left\lfloor \frac{2001}{4}\right\rfloor - \left\lfloor \frac{2001}{3 \times 4}\right\rfloor - \left\lfloor \frac{2001}{3 \times 4}\right\rfloor = {835}. \]
问题20。答案:(B)。
设圆 \( A \) 表示能被3整除的数集,圆 \( B \) 表示能被5整除的数集,圆 \( C \) 表示能被4整除的数集。
我们的任务是求出下图中的阴影区域。
为求阴影区域,我们先求集合 \( A, B \) 与 \( C \) 的并集,再从集合 \( B \) 中减去该并集,得到最终结果。
\[ \left\lfloor \frac{2016}{3}\right\rfloor + \left\lfloor \frac{2016}{4}\right\rfloor + \left\lfloor \frac{2016}{5}\right\rfloor - \left\lfloor \frac{2016}{3 \times 4}\right\rfloor - \left\lfloor \frac{2016}{3 \times 5}\right\rfloor - \left\lfloor \frac{2016}{4 \times 5}\right\rfloor + \left\lfloor \frac{2016}{3 \times 4 \times 5}\right\rfloor - \left\lfloor \frac{2016}{5}\right\rfloor \]
\[ = \left\lfloor \frac{2016}{3}\right\rfloor + \left\lfloor \frac{2016}{4}\right\rfloor - \left\lfloor \frac{2016}{3 \times 4}\right\rfloor - \left\lfloor \frac{2016}{3 \times 5}\right\rfloor - \left\lfloor \frac{2016}{4 \times 5}\right\rfloor + \left\lfloor \frac{2016}{3 \times 4 \times 5}\right\rfloor . \]
\[ = {672} + {504} - {168} - {134} - {100} + {33} = {807}\text{.} \]
问题21。答案:(E)。
设 \( A \) 为3的倍数的整数个数。
设 \( B \) 为5的倍数的整数个数。
设 \( C \) 为15的倍数的整数个数。
答案为 \( n\left( {A \cup B \cup C}\right) - n\left( C\right) \)
\[ n\left( {A \cup B \cup C}\right) = n\left( A\right) + n\left( B\right) + - n\left( {A \cap B}\right) - n\left( {B \cap C}\right) - n\left( {C \cap A}\right) + n\left( {A \cap B \cap C}\right) \]
\[ \left\lfloor \frac{2015}{3}\right\rfloor + \left\lfloor \frac{2015}{5}\right\rfloor + \left\lfloor \frac{2015}{15}\right\rfloor - \left\lfloor \frac{2015}{\mathrm{{lcm}}\left( {3,5}\right) }\right\rfloor - \left\lfloor \frac{2015}{\mathrm{{lcm}}\left( {3,{15}}\right) }\right\rfloor - \left\lfloor \frac{2015}{\mathrm{{lcm}}\left( {5,{15}}\right) }\right\rfloor + \left\lfloor \frac{2015}{\mathrm{{lcm}}\left( {3,5,{15}}\right) }\right\rfloor \]
\[ = \left\lfloor \frac{2015}{3}\right\rfloor + \left\lfloor \frac{2015}{5}\right\rfloor + \left\lfloor \frac{2015}{15}\right\rfloor - \left\lfloor \frac{2015}{15}\right\rfloor - \left\lfloor \frac{2015}{15}\right\rfloor - \left\lfloor \frac{2015}{15}\right\rfloor + \left\lfloor \frac{2015}{15}\right\rfloor \]
\[ = \left\lfloor \frac{2015}{3}\right\rfloor + \left\lfloor \frac{2015}{5}\right\rfloor - \left\lfloor \frac{2015}{15}\right\rfloor \]
\[ n\left( {A \cup B \cup C}\right) - n\left( C\right) = \left\lfloor \frac{2015}{3}\right\rfloor + \left\lfloor \frac{2015}{5}\right\rfloor - 2 \times \left\lfloor \frac{2015}{15}\right\rfloor = {671} + {403} - 2 \times {134} = {806}. \]
问题22。答案:(B)。
\[ \left\lfloor \frac{300}{3}\right\rfloor + \left\lfloor \frac{300}{4}\right\rfloor + \left\lfloor \frac{300}{15}\right\rfloor - \left\lfloor \frac{300}{3 \times 4}\right\rfloor - \left\lfloor \frac{300}{{lcm}\left( {3 \times {15}}\right) }\right\rfloor - \left\lfloor \frac{300}{4 \times {15}}\right\rfloor + \left\lfloor \frac{300}{{lcm}\left( {3 \times 4 \times {15}}\right) }\right\rfloor \]
\[ = {100} + {75} + {20} - {25} - 7 - 5 + 5 = {163} \]
\[ \left\lfloor \frac{300}{15}\right\rfloor = {20} \]
\[ {163} - {20} = {143}\text{.} \]
问题23。答案:(C)。
方法一(官方解法):
在 \( {1000}/2 = {500} \) 个能被2整除的整数中,有 \( \left\lfloor \frac{1000}{6}\right\rfloor = {166} \) 个同时能被2和3整除。因此有 \( {500} - {166} = {334} \) 个能被2整除但不能被3整除,以及 \( {334}/{1000} = {167}/{500} \) 。
方法二:
我们分数的分母显然是1000,即集合中整数的总数。为确定分子,必须找出1到1000中能被2整除且不能被3整除的整数个数。
设 \( A \) 为1到1000中能被2整除的整数集合, \( B \) 为1到1000中能被3整除的整数集合。
我们的任务是求下图中的阴影区域面积:
为求阴影面积,首先计算 \( {AB} \) 的并集
\( n\left( {A \cup B}\right) = n\left( A\right) + n\left( B\right) - n\left( {A \cap B}\right) \)
\[ = \left\lfloor \frac{1000}{2}\right\rfloor + \left\lfloor \frac{1000}{3}\right\rfloor - \left\lfloor \frac{1000}{2 \times 3}\right\rfloor = {500} + {333} - {166} = {667}. \]
接着,减去集合 \( B : {667} - \left\lfloor \frac{1000}{3}\right\rfloor = {667} - {333} = {334} \) 。
因此共有334个能被2整除且不能被3整除的整数。
最终答案即为 \( \frac{334}{1000} = \frac{167}{500} \) 。
问题24。答案:(A)。
设圆A表示能被 \( {15}\left( {3 \times 5}\right) \) 整除的数集,圆B表示能被4整除的数集,圆C表示能被7整除的数集。
我们的任务是求下图中的阴影区域面积:步骤如下:
步骤1:使用容斥原理(PIE)计算并集:
\[ n\left( {A \cup B \cup C}\right) = n\left( A\right) + n\left( B\right) + n\left( C\right) - n\left( {A \cap B}\right) - n\left( {B \cap C}\right) - n\left( {C \cap A}\right) + n\left( {A \cap B \cap C}\right) \]
\[ \left\lfloor \frac{840}{15}\right\rfloor + \left\lfloor \frac{840}{4}\right\rfloor + \left\lfloor \frac{840}{7}\right\rfloor - \left\lfloor \frac{840}{{15} \times 4}\right\rfloor - \left\lfloor \frac{840}{{15} \times 7}\right\rfloor - \left\lfloor \frac{840}{4 \times 7}\right\rfloor + \left\lfloor \frac{840}{{15} \times 4 \times 7}\right\rfloor \]
\[ = {56} + {210} + {120} - {14} - 8 - {30} + 2 = {336} \]
步骤2:计算(B或C)。
\[ n\left( {B \cup C}\right) = n\left( B\right) + n\left( C\right) - n\left( {B \cap C}\right) \]
\[ = {210} + {120} - {30} = {300} \]
步骤3: \( \left( 1\right) - \left( 2\right) \) 即为所求答案,其值为 \( {336} - {300} = {36} \) 。
问题25。答案:(C)。
方法1:
设 \( {a}_{2009} = n \) 。
\[ {2009} = n - n\left( {A \cup B}\right) + n\left( {A \cap C}\right) + n\left( {B \cap C}\right) - n\left( {A \cap B \cap C}\right) \]
\[ = n - \left\lfloor \frac{n}{3}\right\rfloor - \left\lfloor \frac{n}{4}\right\rfloor + \left\lfloor \frac{n}{3 \times 4}\right\rfloor + \left\lfloor \frac{n}{3 \times 5}\right\rfloor + \left\lfloor \frac{n}{4 \times 5}\right\rfloor - \left\lfloor \frac{n}{3 \times 4 \times 5}\right\rfloor \tag{1} \]
我们知道 \( x - 1 < \lfloor x\rfloor \leq x \) 。
\[ \text{So}{2009} < n - \left( {\frac{n}{3} - 1}\right) - \left( {\frac{n}{4} - 1}\right) + \frac{n}{3 \times 4} + \frac{n}{3 \times 5} + \frac{n}{4 \times 5} - \left( {\frac{n}{3 \times 4 \times 5} - 1}\right) = \frac{3}{5}n + 3\text{(} \tag{2} \]
以及 \( {2009} > n - \frac{n}{3} - \frac{n}{4} + \left( {\frac{n}{3 \times 4} - 1}\right) + \left( {\frac{n}{3 \times 5} - 1}\right) + \left( {\frac{n}{4 \times 5} - 1}\right) - \frac{n}{3 \times 4 \times 5} = \frac{3}{5}n - 3 \) (3)
由(2)和(3)可得 \( {3343}\frac{1}{3} < n < {3353}\frac{1}{3} \) 或 \( {3344} \leq n \leq {3353} \) 。
我们知道 \( n \) 既不是3的倍数也不是4的倍数(除非同时是5的倍数),因此 \( n \) 只能是3345、3346、3347、3349、3350、3353之一。
将这些可能值代入(1),我们发现唯一解为 \( n = \) 3347。
方法二:
3、4、5的最小公倍数是60。
\[ {60} - \left\lfloor \frac{60}{3}\right\rfloor - \left\lfloor \frac{60}{4}\right\rfloor + \left\lfloor \frac{60}{3 \times 4}\right\rfloor + \left\lfloor \frac{60}{3 \times 5}\right\rfloor + \left\lfloor \frac{60}{4 \times 5}\right\rfloor - \left\lfloor \frac{60}{3 \times 4 \times 5}\right\rfloor = {36}. \]
在1到60之间, \( {a}_{n} \) 共有36项。
\( {a}_{1} = 1,{a}_{2} = 2,{a}_{3} = 5,{a}_{4} = 7,\ldots ,{a}_{29} = {47},\ldots ,{a}_{36} = {60} \) .
我们知道 \( {2009} = {36} \times {55} + {29} \) 。因此 \( {a}_{2009} = {60} \times {55} + {a}_{29} = {60} \times {55} + {47} = \) 3347。
问题26。解答:(D)。
\( {105} = 3 \times 5 \times 7. \)
设 \( {a}_{1000} = n \) 。
\( {1000} = n - \left\lfloor \frac{n}{3}\right\rfloor - \left\lfloor \frac{n}{5}\right\rfloor - \left\lfloor \frac{n}{7}\right\rfloor + \left\lfloor \frac{n}{3 \times 5}\right\rfloor + \left\lfloor \frac{n}{3 \times 7}\right\rfloor + \left\lfloor \frac{n}{5 \times 7}\right\rfloor - \left\lfloor \frac{n}{3 \times 5 \times 7}\right\rfloor \) (1)
我们知道 \( x - 1 < \lfloor x\rfloor \leq x \) 。
于是 \( {1000} < n - \left( {\frac{n}{3} - 1}\right) - \left( {\frac{n}{5} - 1}\right) - \left( {\frac{n}{7} - 1}\right) + \frac{n}{3 \times 5} + \frac{n}{3 \times 7} + \frac{n}{5 \times 7} - \left( {\frac{n}{3 \times 5 \times 7} - 1}\right) \) (2)
且 \( {1000} > n - \frac{n}{3} - \frac{n}{5} - \frac{n}{7} + \left( {\frac{n}{3 \times 5} - 1}\right) + \left( {\frac{n}{3 \times 7} - 1}\right) + \left( {\frac{n}{5 \times 7} - 1}\right) - \frac{n}{3 \times 4 \times 5} \) (3)
由(2)和(3)可得 \( {2178}\frac{3}{4} < n < {2194}\frac{1}{16} \) 或 \( {3344} \leq n \leq {3353} \) 。
我们知道 \( n \) 与 \( \mathrm{t}{105} \) 互质,因此 \( n \) 只能是2176、2182、2183、2186、2188、2189、2192、2194之一。
将这些可能值代入(1),我们发现唯一解为 \( n = \) 2186。
方法二:
\( {105} = 3 \times 5 \times 7 \)
从1到105中与105互质的项的个数为
\( {105} - \left\lfloor \frac{105}{3}\right\rfloor - \left\lfloor \frac{105}{5}\right\rfloor - \left\lfloor \frac{105}{7}\right\rfloor + \left\lfloor \frac{105}{3 \times 5}\right\rfloor + \left\lfloor \frac{105}{3 \times 7}\right\rfloor + \left\lfloor \frac{105}{5 \times 7}\right\rfloor - \left\lfloor \frac{105}{3 \times 5 \times 7}\right\rfloor \)
\( = {105} - \left( {{35} + {21} + {15}}\right) + \left( {7 + 3 + 5}\right) - 1 = {48} \) .
因此 \( {a}_{1} = 1,{a}_{2} = 2,{a}_{3} = 4,\ldots ,{a}_{40} = {86},{a}_{41} = {88},{a}_{42} = {89},{a}_{43} = {92},{a}_{44} = {94},{a}_{45} = \) \( {97},{a}_{46} = {101},{a}_{47} = {103} \) ,且 \( {a}_{48} = {104} \) 。
我们还有 \( {a}_{49} = {105} + 1,{a}_{50} = {105} + 2,{a}_{51} = {105} + 4,\ldots ,{a}_{96} = {105} + {104},\ldots \)
由于 \( {1000} = {48} \times {20} + {40} \) ,所以 \( {a}_{1000} = {105} \times {55} + {a}_{40} = {105} \times {55} + {86} = {2186} \) 。